the crankshaft in a race car goes from rest to 3600 rpm rpm in 2.8 s.
What is the crankshaft's angular acceleration?
How many revolutions does it make while reaching 3600 rpm?

Answers

Answer 1

The angular acceleration of the crankshaft can be found using the formula:
angular acceleration = (final angular velocity - initial angular velocity) / time

The initial angular velocity is 0 since the crankshaft starts from rest. The final angular velocity can be found by converting 3600 rpm to radians per second:

final angular velocity = (3600 rpm) x (2π radians/1 revolution) x (1 min/60 s) = 377 radians/s

Plugging in the values, we get:
angular acceleration = (377 radians/s - 0 radians/s) / 2.8 s = 134.6 radians/s^2

Therefore, the angular acceleration of the crankshaft is 134.6 radians/s^2.

To find the number of revolutions the crankshaft makes while reaching 3600 rpm, we can use the formula:
number of revolutions = final angular velocity / (2π radians/1 revolution)

Plugging in the values, we get:
number of revolutions = 377 radians/s / (2π radians/1 revolution) = 59.9 revolutions

Therefore, the crankshaft makes approximately 59.9 revolutions while reaching 3600 rpm.

To know more about angular acceleration :

https://brainly.com/question/30787003

#SPJ11


Related Questions

5 of 225 of 22 Items
12:41







Question
The basic concept of how a simple motor works is explained by which statement?

Answers

Answer:

The basic concept of how a simple motor works is that you put electricity into it at one end and an axle (metal rod) rotates at the other end giving you the power to drive a machine of some kind. The simple motors you see explained in science books are based on a piece of wire bent into a rectangular loop, which is suspended between the poles of a magnet. In order for a motor to run on AC, it requires two winding magnets that don’t touch. They move the motor through a phenomenon known as induction.

I hope this helps! Let me know if I'm wrong!

Explanation:

calculate the current flowing through the heater and determine how much power the heater will provide

Answers

In the below given conditions the current flowing through the heater would be 12 amps and power provided by the heater would be 1440 watts.

To calculate the current flowing through the heater and determine the power it will provide, we need to know the voltage and resistance of the heater.

Let's assume that the voltage is 120V and the resistance is 10 ohms.

Using Ohm's law, we can calculate the current as I = V/R, which gives us 12 amps.

To determine the power provided by the heater, we can use the formula P = VI, where P is the power, V is the voltage and I is the current.

Substituting the values, we get

P = 120V x 12A = 1440 watts.

Therefore, the heater will provide 1440 watts of power and the current flowing through it will be 12 amps.

It is important to note that these calculations are based on the assumptions made about the voltage and resistance of the heater. Actual measurements may vary and should be taken for accurate results.

For more such answers on Current and power

https://brainly.com/question/19266852

#SPJ11

Students set up an experiment to test the effect of gravity on different objects. They fill balloons with different amounts of water and drop them from the third-floor window of their school. They time how long each balloon takes to reach the ground. They repeat the process three times. What is the experiment’s variable?
answers
number of times the drops were repeated


times that the balloons take to drop

mass of the balloons


height of the window

Answers

The number of times the drops were repeated and the mass of the balloons may be controlled variables that are kept constant during the experiment to isolate the effect of the height of the window on the time it takes for the balloons to reach the ground.

What is Isolated System?

An isolated system is a concept in thermodynamics and physics that refers to a system that does not exchange energy or matter with its surroundings. It is a closed system with respect to both energy and matter, meaning that no energy or matter is transferred across its boundaries. In an isolated system, the total energy, including both kinetic and potential energy, remains constant over time. This is known as the principle of conservation of energy.

The experiment's variable in this case is the height of the window from which the balloons are dropped. The students are specifically testing the effect of gravity, which is influenced by the height from which an object falls. By varying the height of the window, the students are manipulating the independent variable (height of the window) to observe the effect on the dependent variable (time it takes for the balloons to reach the ground).

Learn more about Isolated System from the given link

https://brainly.com/question/2846657

#SPJ1

satellite motion: if earth had twice its present mass but it orbited at the same distance from the sun as it does now, its orbital period would be 1 year 1,5 year 2 year 2,5 year

Answers

The orbital period of the Earth around the Sun is determined by its distance from the Sun and its mass. If the Earth had twice its present mass but orbited at the same distance from the Sun, its gravitational attraction to the Sun would be stronger, resulting in a longer orbital period. Using Kepler's third law of planetary motion, we can calculate the new orbital period as follows:

T^2 = (4π^2/G) x (r^3/m)

where T is the orbital period, G is the gravitational constant, r is the distance from the Earth to the Sun, and m is the mass of the Earth.

Plugging in the values, we get:

T^2 = (4π^2/6.6743 x 10^-11) x [(149.6 x 10^6)^3 / (2 x 5.9722 x 10^24)]
T^2 = 1.085 x 10^20
T = √(1.085 x 10^20)
T = 1.09 x 10^10 seconds

Converting this to years, we get:
T = 346 years

Therefore, if the Earth had twice its present mass but orbited at the same distance from the Sun as it does now, its orbital period would be approximately 346 years.

To know more about Kepler's third law :

https://brainly.com/question/30404084

#SPJ11

Newton's first law of motion states that an object in motion will stay in motion unless acted upon by a force.

What are two forces on earth that could change the motion of the object?

Responses

friction and gravity
friction and gravity

speed and acceleration

heat and light

direction and time

Answers

The two forces on Earth that could change the motion of an object are friction and gravity.

Friction is a force that opposes the motion of an object when it is in contact with another surface. It can cause an object to slow down or come to a stop.

Gravity is a force of attraction between two objects, and it can cause an object to accelerate toward the center of the earth or towards another massive object. The gravitational force on an object depends on its mass and the distance between it and the other object.

Speed and acceleration are not forces, but rather measures of motion. Heat and light are also not forces that can change the motion of an object, but rather forms of energy that can be transferred to an object and affect its temperature or behavior. Direction and time are not forces, but concepts related to an object's motion.

Learn More about Newton's first law of motion:

https://brainly.com/question/1222214

GUYS SOMEONE PLEASE HELP. i will give brainliest!!


You and your friends are driving home from the cottage and you want to bring back a camping trailer your parents left there. Before leaving, you have a troubling thought. What if the hitch on your car is not strong enough to pull this trailer? Your car has a Class I hitch which is rated with a pulling strength of 2000 lbs. On the side of the trailer a sticker gives the trailer's weight as 3600 lbs. You know from your driver's manual that your car has a weight of 1800 lbs. The real challenge will be getting on to the highway. The ramp to the highway begins with a stop sign and is about 140 m long on level ground. The highway traffic travels at 100 km/h. Can you safely merge into the flow of traffic? You may assume friction effects are very small.

Answers

You should not attempt to tow the camping trailer with your current car and hitch setup, as the hitch is not strong enough to handle the weight of the trailer.

How to solve

A Class I hitch is rated for a pulling strength of 2000 lbs, while the trailer's weight is 3600 lbs.

This means that the hitch is not strong enough to safely pull the trailer, which poses a significant safety risk.

However, let's assume for a moment that the hitch was strong enough and evaluate if you could safely merge into the flow of traffic.

To assess whether you can safely merge, we need to determine if your car can accelerate to the highway speed of 100 km/h (27.8 m/s) within the 140 m long ramp.

We can use the following kinematic equation to solve for acceleration:

v^2 = u^2 + 2as

Where:

v is the final velocity (100 km/h or 27.8 m/s)

u is the initial velocity (0 m/s, as you start from a stop sign)

a is the acceleration

s is the distance (140 m)

Rearranging the equation to solve for acceleration:

a = (v^2 - u^2) / (2s)

a = (27.8^2 - 0^2) / (2 * 140)

a ≈ 2.77 m/s²

Now, we need to calculate the force required to achieve this acceleration. We'll use Newton's second law:

F = m * a

The total mass of the car and the trailer is 1800 lbs (car) + 3600 lbs (trailer) = 5400 lbs. To convert this to kilograms, we multiply by 0.453592 (1 lb = 0.453592 kg):

5400 lbs * 0.453592 kg/lb ≈ 2449 kg

Now we can calculate the force required:

F = 2449 kg * 2.77 m/s² ≈ 6781 N

Now let's compare this force to the pulling strength of the hitch. The hitch can handle 2000 lbs, which is equivalent to:

2000 lbs * 4.44822 N/lb ≈ 8896 N

In this scenario, the required force to achieve the necessary acceleration (6781 N) is less than the pulling strength of the hitch (8896 N).

However, as mentioned earlier, the hitch is not strong enough to safely pull the trailer due to the trailer's weight exceeding the hitch's rated capacity.

In conclusion, you should not attempt to tow the camping trailer with your current car and hitch setup, as the hitch is not strong enough to handle the weight of the trailer.

Even if the hitch was strong enough, towing a heavy trailer would still pose other challenges and safety risks, such as stopping distance, stability, and maneuverability.

Read more about acceleration here:

https://brainly.com/question/460763

#SPJ1

A disk of radius R = 7. 52 cm is centered at the origin and lies along the y – z plane. The disk has a surface charge density σ = 5. 88 × 10 − 6 C / m 2. Evaluate the electric field produced by this disk along the x axis at point (P = 1. 01 m, 0. 00 m). The Coulomb force constant k = 1 / ( 4 π ϵ 0 ) = 8. 99 × 10 9 N ⋅ m 2 / C 2

Answers

The electric field produced by the disk at point P along the x-axis is approximately 333.89 N/C.

Since the disk lies in the y-z plane, the electric field produced by the disk will only have an x-component, which can be calculated using the formula for the electric field produced by a charged disk:

E = σ / (2ε₀) * [1 - (z / √(R² + z²))]

At point P(1.01 m, 0.00 m), the distance from the disk along the z-axis is z = 0, so the formula reduces to:

E = σ / (2ε₀) = (5.88 × 10^-6 C/m²) / (2 * 8.85 × 10^-12 F/m) ≈ 333.89 N/C

Therefore, the electric field produced by the disk is 333.89 N/C.

To know more about electric field , here

brainly.com/question/15800304

#SPJ4

given the temperature readings in the topmost tube (in the system diagrammed), which would be reasonable temperatures for points 1 to 5 in the lower tube?

Answers

To help you determine the reasonable temperatures for points 1 to 5 in the lower tube, we'll need to consider the given temperature readings in the topmost tube and the temperature changes in the system.

Let's go through the steps to find the temperatures for each point.
Analyze the temperature readings in the topmost tube.
- Observe and record the temperatures at different points in the topmost tube.

Understand the heat transfer process in the system.
- Consider the direction of heat flow, such as from hot to cold regions.

Determine the temperature differences between the tubes.
- Based on the heat transfer process, estimate the temperature differences between the corresponding points in the topmost and lower tubes.

Calculate the temperatures for points 1 to 5 in the lower tube.
- Subtract the estimated temperature differences from the temperatures of the corresponding points in the topmost tube.

By following these steps, you will be able to find the reasonable temperatures for points 1 to 5 in the lower tube based on the given temperature readings in the topmost tube.

*complete question: Given the temperature readings in a topmost tube, which would be reasonable temperatures for points 1 to 5 in the lower tube?

More on temperature: https://brainly.com/question/26228909

#SPJ11

particles of various masses, charges, and speeds are injected into a region in which a uniform field and a uniform field are perpendicular to each other. all the particles are initially moving in the same direction. which two conditions must be simultaneously fulfilled for the particles to continue moving in a straight line after entering the region?

Answers

The two conditions that must be simultaneously fulfilled for the particles to continue moving in a straight line after entering the region are:

1. The direction of the initial velocity of the particles must be perpendicular to both the magnetic and electric fields.
2. The Lorentz force experienced by the particles due to the magnetic and electric fields must be equal and opposite to the initial force that caused the particles to move in the first place.

The Lorentz force is given by the equation F = q(E + v x B), where F is the force, q is the charge of the particle, E is the electric field, v is the velocity of the particle, and B is the magnetic field. If the Lorentz force is equal and opposite to the initial force, then the particles will continue to move in a straight line.

part a what is the highest order that contains the entire visible spectrum? (the wavelength range of the visible spectrum is approximately 400-700 nm .)

Answers

The highest order that contains the entire visible spectrum is the first order. The visible spectrum is the range of wavelengths of light that are visible to the human eye.

The first order is the smallest wavelength range that contains the entire visible spectrum, which ranges from approximately 400-700 nm.

This is because the visible spectrum is a relatively small range of the electromagnetic spectrum compared to other regions, such as radio waves or X-rays.

When light is diffracted through a diffraction grating, the first order is the most commonly used order as it contains the majority of the visible spectrum.

However, higher orders can also contain parts of the visible spectrum, but they are less commonly used as they contain smaller ranges of wavelengths.

For more such answers on the Visible spectrum

https://brainly.com/question/9839674

#SPJ11

how can sonar best be used to monitor the hydrosphere

Answers

Sonar can be a useful tool for monitoring the hydrosphere, which includes all of the water on and beneath the Earth's surface.

Sonar works by emitting sound waves that bounce off objects in the water, and then measuring the time it takes for the sound waves to return to the source. By analyzing the echoes, scientists can map the seafloor, measure the depth of the water, and even identify the size and location of marine organisms.

Sonar can also be used to monitor the movements of water masses, including ocean currents, tides, and storm surges. This information is important for understanding global climate patterns and predicting the effects of natural disasters

Learn more about global climate

https://brainly.com/question/27919422

#SPJ4

which force pairs must be equal because they are action/reaction pairs?multiple select question.the upward normal force on a car and the downward push of the car on the ground.the rightward force of a pull on a box traveling at constant velocity and the leftward friction force.the upward normal force on a book at rest on the table and its downward weight.the rightward force of you pushing on a wall and the leftward force of the wall pushing on you.

Answers

Force pairs that must be equal because they are action/reaction pairs. According to Newton's Third Law of Motion, for every action, there is an equal and opposite reaction. This means that action and reaction forces are always equal in magnitude but opposite in direction.

Some examples of action/reaction force pairs include:
1. When you push a book across a table (action), the book pushes back with an equal force (reaction).
2. When a person jumps off a diving board (action), the diving board exerts an equal and opposite force on the person (reaction).
3. A person walking on the ground pushes against the ground (action), and the ground pushes back with an equal force (reaction).

In all these cases, the action/reaction force pairs are equal and opposite, illustrating Newton's Third Law of Motion.

To know more about Newton's Third Law of Motion:

https://brainly.com/question/29768600

#SPJ11

consider example 3.9 on page 87. suppose that the dog runs at a speed of 7.0 m/s. how far does the dog travel horizontally from the edge of the dock before splashing down?

Answers

The dog travels 2.1 meters horizontally from the edge of the dock before splashing down.

When dog runs at speed 7.0m/s,how far he reached the edge of the dock before splashing down?

We know that the distance the dog travels horizontally before splashing down is equal to the product of the time in the air and the horizontal velocity of the dog.

Using the equation: distance = velocity x time

We can first solve for the time in the air.

The initial vertical velocity of the dog is zero, and we can use the equation:

distance = 1/2 x acceleration x time⁻²

to find the time it takes for the dog to fall from the edge of the dock to the water.

Assuming a gravitational acceleration of 9.8 m/s⁻², we get:

distance = 1/2 x 9.8 m/s⁻² x time⁻²

0.91 meters = 4.9 x time⁻²

time = sqrt(0.91 / 4.9) = 0.3 seconds

Now that we know the time in the air, we can find the horizontal distance traveled by the dog before splashing down.

Using the equation:

distance = velocity x time

where velocity is the horizontal velocity of the dog, which we know is 7.0 m/s, we get:

distance = 7.0 m/s x 0.3 s = 2.1 meters

The dog travels 2.1 meters horizontally from the edge of the dock before splashing down.

Learn more about Speed and splashing down

brainly.com/question/14575052

#SPJ11

a bicycle travels from a to b. half the time it travels with speed 20 km/h, and half the time with the speed 30 km/h. what is the average speed?

Answers

The bicycle travels from a to b. half the time it travels with speed 20 km/h, and half the time with the speed 30 km/h, therefore the average speed of the bicycle is 25 km/h.

To find the average speed of the bicycle, we need to use the formula:

Average Speed = Total Distance / Total Time

Since we don't know the distance between points A and B, we can assume it to be 'd' kilometers.

Let's say the time taken by the bicycle to travel from A to B is 't' hours.

According to the problem statement, the bicycle travels at 20 km/h for half the time and 30 km/h for the other half. This means that it covers the first half of the distance at 20 km/h and the second half at 30 km/h.

Hence, the time taken to cover the first half of the distance is (t/2) hours, and the time taken to cover the second half is also (t/2) hours.

Now, we can calculate the total time taken by the bicycle as follows:

Total Time = (t/2) + (t/2) = t hours

Next, we can calculate the total distance traveled by the bicycle as follows:

Total Distance = Distance Covered in First Half + Distance Covered in Second Half
                = (20 km/h) x (t/2) + (30 km/h) x (t/2)
                = 25t km

Substituting these values in the formula for average speed, we get:

Average Speed = Total Distance / Total Time
                   = 25t km / t hours
                   = 25 km/h

Therefore, the average speed of the bicycle is 25 km/h.

More on speed: https://brainly.com/question/15586792

#SPJ11

if a plunge pool undercuts the support of the resistant rock layer above and causes it to collapse, then the

Answers

If a plunge pool undercuts the support of the resistant rock layer above and causes it to collapse, then this can result in a potentially dangerous situation, the collapse can cause erosion of the surrounding soil and vegetation, leading to further instability of the area.

The collapse of the resistant rock layer can lead to a landslide or rockfall, which can cause significant damage to the surrounding area and pose a threat to anyone in the vicinity. Additionally, the collapse can cause erosion of the surrounding soil and vegetation, leading to further instability of the area.

To prevent such occurrences, it is important to properly design and maintain plunge pools. The proper design includes ensuring that the pool is not located near a resistant rock layer or if it is, that measures are put in place to prevent the pool from undercutting the rock.

This may include reinforcing the rock layer, installing retaining walls or other support structures, or moving the pool to a different location.

Regular maintenance of the plunge pool is also crucial to prevent erosion and undercutting of the rock layer. This may involve monitoring the pool for signs of erosion or instability and taking corrective action if necessary, such as repairing or reinforcing the surrounding area.

Overall, it is important to ensure that plunge pools are designed and maintained properly to prevent the undercutting of resistant rock layers and potential collapses, which can have serious consequences.

To know more about erosion refer here:

https://brainly.com/question/1599677#

#SPJ11

a particle with a cahrge of 1 c is moving at 45 angle with respect to the positive x axis in teh horizontal xy-plane. the velocity of the charge is 1 m/s. a magnetic field of 1 t is directed in the negative x direction. what is the magnetic force acting on the charge?

Answers

The magnetic force acting on the charged particle is -0.707 N in the k direction and 0.707 N in the j direction.

In this problem, the charge of the particle is given as 1 C, and the velocity of the particle is 1 m/s at an angle of 45 degrees to the positive x-axis. We can break down the velocity vector into its x and y components as follows:

vx = vcos(45) = 0.707 m/s

vy = vsin(45) = 0.707 m/s

The magnetic field is given as 1 T in the negative x direction.

Substituting these values into the formula for the magnetic force, we get:

F = q * (vxi + vyj + 0k) x (-Bi)

where I, j, and k are the unit vectors in the x, y, and z directions, respectively.

Expanding the cross product, we get:

F = q*(-vxB)k + qvyB*j

Substituting the values for q, vx, vy, and B, we get:

F = (1 C) (-0.707 m/s) (1 T) k + (1 C) (0.707 m/s) *(1 T) *j

Simplifying, we get:

F = -0.707 k + 0.707 j

To know more about Charge:

https://brainly.com/question/3412043

#SPJ4

what is the average magnitude of the poynting vector 4.50 mi from a radio transmitter broadcasting isotropically (equally in all directions) with an average power of 200 kw?

Answers

The average magnitude of the Poynting vector at a distance of 4.50 miles from the transmitter is approximately 40.8 nanowatts per square meter.

This problem is about finding the average magnitude of the Poynting vector, which is a measure of the energy flow of electromagnetic waves, at a distance of 4.50 miles from an isotropic radio transmitter.

The transmitter broadcasts equally in all directions with an average power of 200 kW. We can use a formula that relates the power density of the transmitter to the Poynting vector. By substituting the given values and using the speed of light as the propagation velocity of electromagnetic waves, we can calculate the Poynting vector.

The average magnitude of the Poynting vector at a distance of 4.50 miles from the transmitter is approximately 40.8 nanowatts per square meter.

Learn more about magnitude:

https://brainly.com/question/30881682

#SPJ4

For a satellite to orbit Earth at a constant distance, its centrifugal acceleration must be equal and opposite Earth's gravitational acceleration. If a satellite is to orbit at a constant distance from Earth at a circular radius of 8,000,000 m, what is the required velocity of the satellite? (Assume the acceleration due to Earth's gravity is 6.2 m/s2 at this altitude.).

Answers

The required velocity of the satellite to orbit Earth at a constant distance of 8,000,000 m is 7,905 m/s.

What is Gravity?

Gravity is a force that attracts two bodies with mass towards each other. It is one of the four fundamental forces of nature and is responsible for holding planets in orbit around stars and stars in orbit around galaxies. Gravity is described by Einstein's theory of general relativity, which states that gravity is the result of the curvature of spacetime caused by the presence of mass or energy.

where G is the gravitational constant M is the mass of the Earth  and r is the distance between the satellite and the center of the Earth (8,000,000 m).

First, we need to calculate the gravitational acceleration due to the Earth's gravity at this altitude using the formula:

g = GM/[tex]r^{2}[/tex]

g = (6.67 x 10^-11 N [tex]m^{2}[/tex]/[tex]kg^{2}[/tex]) x (5.97 x [tex]10^{24}[/tex] kg) / (8,000,000 m)^2

g = 6.19 m/[tex]s^{2}[/tex]

The required velocity can be found using:

v = √(GM/r)

v = √[(6.67 x 10^-11 N[tex]m^{2}[/tex]/[tex]kg^{2}[/tex]) x (5.97 x [tex]10^{24}[/tex] kg) / (8,000,000 m)]

v = 7,905 m/s

Learn more about Gravity from the given link

https://brainly.com/question/940770

#SPJ1

A 500 N force is applied to a 25m/s2 object. The mass of the object is ____.

A. ) 20kg
B. ) 20 m/s
C. ) 12,500 m/s
D. ) 12,500 kg​

Answers

A 500 N force is applied to a 25m/s2 object. The mass of the object is 20kg. The correct answer is option: A.

The force applied to an object is related to its mass and acceleration through the equation:

F = ma,

where F is the force, m is the mass, and a is the acceleration. Rearranging this equation, we get:

m = F/a.

In the given problem, a force of 500 N is applied to the object, and its acceleration is 25 m/s^2.

Substituting these values in the formula, we get :

m = 500 N / 25 m/s^2 = 20 kg.

Therefore, the mass of the object is 20 kg, which is option A.

To know more about acceleration, here

brainly.com/question/12550364

#SPJ4

starting from rest, a disk rotates about its central axis with constant angular acceleration. in 5.0 s, it rotates 50 rad. what is the instantaneous angular velocity of the disk at the end of the 20.0 s?

Answers

The instantaneous angular velocity is 20.0 s is 400 rad/s.

What is the final instantaneous angular velocity of a disk rotating about its central axis with constant angular acceleration?

Since the angular acceleration is constant, we can use the formula:

[tex]θ = 1/2 * α * t^2 + ω0 * t[/tex]

where

[tex]θ = angle rotated = 50 rad[/tex]

[tex]α = angular acceleration[/tex]

[tex]t = time = 5.0 s[/tex]

[tex]ω0 = initial angular velocity = 0 (starting from rest)[/tex]

Solving for α, we get:

[tex]α = 2 * (θ - ω0 * t) / t^2 = 2 * 50 rad / 5.0 s^2 = 20 rad/s^2[/tex]

Now, using the formula:

[tex]ω = α * t + ω0[/tex]

where

ω = instantaneous angular velocity at the end of 20.0 s (what we need to find)

[tex]α = angular acceleration = 20 rad/s^2[/tex]

[tex]t = time = 20.0 s[/tex]

[tex]ω0 = initial angular velocity = 0 (starting from rest)[/tex]

we get:

[tex]ω = 20 rad/s^2 * 20.0 s + 0 = 400 rad/s[/tex]

Therefore, the instantaneous angular velocity of the disk at the end of 20.0 s is 400 rad/s.

Learn more about angular acceleration

brainly.com/question/29428475

#SPJ11

how fast is moving light that has traveled through transparent glass and exited on the opposite side with respect to the incoming speed?

Answers

The light that has traveled through transparent glass and exited on the opposite side will move at the same speed as it was moving before entering the glass, but it would have traveled slower while inside the glass.

The speed of light changes when it travels through a transparent medium like glass. The speed of light in vacuum or air is approximately 299,792,458 meters per second (often rounded to 3.00 x 10⁸ m/s), but it slows down when it passes through a medium like glass. The amount of slowing down depends on the refractive index of the material, which is a measure of how much the speed of light is reduced as it passes through the material.

For typical glasses, the refractive index is around 1.5, which means that the speed of light is reduced by a factor of about 1.5 when it passes through the glass. So, if the speed of light in vacuum or air is taken as 1, the speed of light in glass would be approximately 2/3 (or 0.67) of its original speed.

When the light exits the glass on the opposite side, it returns to its original speed in air or vacuum. Therefore, the light exits the glass with the same speed it had before it entered the glass, as long as it is not absorbed or scattered by the glass.

Learn more about the refractive index:

https://brainly.com/question/83184

#SPJ11

a piece of wood is has a density of 0.600 g/cm3. when dipped in water, what fraction of the wood is submerged inside water? (give answer to 3 significant figures)

Answers

To determine the fraction of the wood submerged in water, we need to compare the density of the wood to the density of water.

The density of water is 1.000 g/cm3 at standard temperature and pressure.

If the wood has a density of 0.600 g/cm3, it is less dense than water, which means it will float on water.

To determine the fraction of the wood submerged in water, we can use the following formula:

fraction submerged = (volume submerged) / (total volume)

Since the wood floats on water, the volume of water displaced by the wood is equal to the volume of the submerged portion of the wood.

The total volume of the wood is equal to its mass divided by its density:

total volume = mass / density

We don't have the mass of the wood, but we can use any arbitrary value to determine the fraction submerged.

Let's assume the wood has a mass of 100 g.

total volume = mass / density = 100 g / 0.600 g/cm3 = 166.67 cm3

Now, let's assume that when the wood is submerged in water, it displaces 80 cm3 of water.

fraction submerged = (volume submerged) / (total volume) = 80 cm3 / 166.67 cm3 = 0.48

Therefore, approximately 48% of the wood is submerged in water.

To know more about fraction visit link :

https://brainly.com/question/31215473

#SPJ11

a crane uses a block and tackle to lift a 2000 N flagstone to a height of 30 m. How much work is done on the flagstone

Answers

The work done on the flagstone by the crane is equal to the product of the force applied and the distance it is moved in the direction of the force. In this case, the force is the tension in the rope of the block and tackle that is lifting the flagstone, and the distance is the height to which the flagstone is lifted.

The block and tackle system reduces the force required to lift the flagstone, while increasing the distance through which the rope must be pulled. The amount of force reduction depends on the number of pulleys in the system. Assuming that the block and tackle has a mechanical advantage of 4 (meaning that the force required to lift the flagstone is 1/4 of the weight of the flagstone), the tension in the rope will be:

Tension = Force required / Mechanical advantage
Tension = 2000 N / 4
Tension = 500 N

The work done on the flagstone by the crane is then:

Work = Force x Distance
Work = 500 N x 30 m
Work = 15,000 J

Therefore, the crane does 15,000 joules of work on the flagstone.

What does it mean when we say our sense of motion depends on our frame of reference? Include the phrases “fixed frame” and “moving frame” in your answer.

Answers

frame of reference that is not inertial. A non-inertial frame is now defined as a frame that accelerates relative to the underlying inertial reference frame. Newton's law won't be valid.

How does the framework function?

Performance could change depending on the lighting. The Frame automatically modifies the Plasma tvs brightness and contrasting settings after analyzing the lighting conditions in the room and the light level of your content.

What distinguishes a system from a frame?

the hard architecture (bones and condyle) that serves as an animal's body's framework. skeletal system, skeleton, and systema skeletale. system: a collection of organs or bodily parts that function or are anatomically related; "the body contains a system for organs for digestion."

To know more about frame visit:

https://brainly.com/question/9708057

#SPJ1

describe the relationship between the angle of incoming light and the percentage of light that is transmitted through the water versus the percentage of light that is reflected

Answers

The angle of incoming light plays a significant role in determining the percentage of light that is transmitted through water versus the percentage of light that is reflected. As the angle of incidence of light increases, the amount of light that is transmitted through the water decreases, while the amount of light that is reflected off the surface of the water increases.

This is due to the fact that at higher angles of incidence, the light has to travel through more water, which causes it to be absorbed and scattered more, leading to a decrease in the amount of transmitted light. Additionally, the angle of incidence also affects the polarization of the reflected light, which can impact the amount of light that is reflected.Overall, the relationship between the angle of incoming light and the percentage of light that is transmitted versus reflected is complex and depends on a variety of factors, including the properties of the water, the characteristics of the light, and the angle of incidence.

To learn more about angle of incidence please visit:

https://brainly.com/question/30048990

#SPJ11

how fast would a 67 kg person have to run to have that amount of energy? express your answer with the appropriate units.

Answers

The 67 kg person has to run at the speed of  [tex]\sqrt{((2 \times KE) / 67)}[/tex] m/s to have that amount of energy.

To calculate the kinetic energy (KE) of a person running, we can use the formula

KE = [tex]0.5 \times m \times v^2[/tex]

where m is the mass (67 kg) and v is the velocity in meters per second (m/s).

First, we need to determine the desired amount of energy, which is not provided in the question.
Obtain the desired energy value (in joules, J) you want the person to have while running.

Plug the mass (67 kg) and the desired energy value into the KE formula: [tex]KE = 0.5 \times 67 \times v^2[/tex].

Rearrange the formula to isolate v:

[tex]v^2 = (2 \times KE) / 67[/tex] m/s

Calculate v by taking the square root

[tex]v = \sqrt {(2 \times KE) / 67)}[/tex] m/s

The calculated value of v will be the velocity in meters per second (m/s) required for the person to have the desired amount of energy while running.
Remember to replace "KE" with the desired energy value in joules, and you will get the velocity (in m/s) needed for a 67 kg person to have that amount of energy.

For more such answers on Kinetic Energy

https://brainly.com/question/25959744

#SPJ11

A 20 Kg bike accelerates at 10 m/s^2. With what force was the person pedaling

Answers

To calculate the force with which the person is pedaling, we need to use Newton's second law of motion, which states that the net force acting on an object is equal to the mass of the object times its acceleration. In this case, the net force is the force applied by the person pedaling the bike. Therefore, we have:

Net force = mass x acceleration

Substituting the given values, we get:

Net force = 20 kg x 10 m/s^2

Net force = 200 N

Therefore, the force with which the person is pedaling the bike is 200 N.

-The diagram above represents two identical pulses approaching each other in a uniform medium.
As the pulses meet and are superposed, the maximum displacement of the medium is?
- 6 cm
0 cm
6 cm
3 cm

Answers

On the basis of constructive interference, when two identical pulses go together in a homogeneous medium and the pulses meet and overlap, the maximum displacement of the medium is equal to 6 cm. So, option (c) is right.

Wave interference is the phenomenon where two waves meet while propagating in the same medium. Constructive interference is a form of interference. It takes place when two pulses meet each other and form a larger pulse. The amplitude of the resulting larger pulse is the sum of the amplitudes of the first two pulses.

This could be done at meetings of two crests or troughs. It can appear anywhere between the two interfering waves are displaced upward. But the two negative effects are also seen when they move downwards.This is shown in the image above. Since we have two identical wave pluses, they are close together in a uniform medium.

Now, Amplitude of pluse A = 3 cm

Amplitude of pluse B = 3 cm

So, the pulses meet and are superposed, the amplitude or maximum displacement of the medium is sum of amplitudes of pluses, that is 3cm + 3 cm = 6 cm. Therefore, the displacement value should be 6 cm.

For more information about Constructive interference, visit :

https://brainly.com/question/11808546

#SPJ4

Complete question:

-The diagram above represents two identical pulses approaching each other in a uniform medium.

As the pulses meet and are superposed, the maximum displacement of the medium is?

a) - 6 cm

b) 0 cm

c)6 cm

d) 3 cm

According to constructive interference, the maximum displacement of the medium when two identical pulses collide and overlap in a homogeneous medium is equal to 6 cm. Option (c) is correct, therefore.

When two waves collide while moving across the same medium, the result is known as wave interference. Interference includes constructive interference. It happens when two pulses collide and create a bigger pulse. The initial two pulses' amplitudes are added to create the larger, resultant pulse.

This could be carried out when two crests or troughs meet. It could show up anywhere where the two competing waves are displaced upward. But when they descend, the two adverse impacts are also evident.In the picture up top, this is evident. In a homogeneous medium, they are close together since we have two identical wave pluses.

The current amplitude of pluse A is 3 cm.

The pluse B's amplitude is 3 cm.

The sum of the plus amplitudes of the pulses, or 3 cm + 3 cm = 6 cm, is the amplitude or maximum displacement of the medium as the pulses collide and superimpose. So, 6 cm should be the displacement value.

learn more about Constructive interference here:

https://brainly.com/question/16098226

#SPJ11

a 3.0 m long rigid beam with a mass of 100 kg is supported at each end. an 80 kg student stands 2.0 m from support 1. how much upward force does each support exert on the beam?

Answers

Answer:

[tex]752.1\; {\rm N}[/tex] from support [tex]\texttt{1}[/tex] ([tex]2.0\; {\rm m}[/tex] from the student.)

[tex]1013.7\; {\rm N}[/tex] from support [tex]\texttt{2}[/tex] ([tex]1.0\; {\rm m}[/tex] from the student.)

(Assuming that [tex]g = 9.81\; {\rm N\cdot kg^{-1}}[/tex], the beam is level with negligible height, and that the density of the beam is uniform.)

Explanation:

Weight of the beam: [tex](100\; {\rm kg})\, (9.81\; {\rm N\cdot kg^{-1}}) = 981\; {\rm N}[/tex].

Weight of the student: [tex](80\; {\rm kg})\, (9.81\; {\rm N\cdot kg^{-1}}) = 784.8\; {\rm N}[/tex].

Assuming that the beam is uniform. The center of mass of the beam will be [tex](1/2)\, (3.0\; {\rm m}) = 1.5\; {\rm m}[/tex] away from each support.

Consider support [tex]\texttt{1}[/tex] as the fulcrum:

For support [tex]\texttt{2}[/tex] (with an upward force of [tex]N_{\texttt{2}}[/tex]), the lever arm is [tex]3.0\; {\rm m}[/tex].For the center of mass of the beam ([tex]981\; {\rm N}[/tex]), the lever arm is [tex]1.5\; {\rm m}[/tex].For the weight of the student ([tex]784.8\; {\rm N}[/tex]), the lever arm is [tex]2.0\; {\rm m}[/tex].

Hence:

[tex]\begin{aligned}N_{\texttt{2}}\, (3.0) = (981)\, (1.5) + (784.8) \, (2.0) \end{aligned}[/tex].

[tex]\begin{aligned}N_{\texttt{2}} &= \frac{(981)\, (1.5) + (784.8) \, (2.0)}{3.0} \; {\rm N} = 1013.7\; {\rm N}\end{aligned}[/tex].

In other words, support [tex]\texttt{2}[/tex] would exert an upward force of [tex]1013.7\; {\rm N}[/tex] on the beam.

Similarly, consider support [tex]\texttt{2}[/tex] as the fulcrum:

For support [tex]\texttt{1}[/tex] (with an upward force of [tex]N_{\texttt{1}}[/tex]), the lever arm is [tex]3.0\; {\rm m}[/tex].For the center of mass of the beam ([tex]981\; {\rm N}[/tex]), the lever arm is [tex]1.5\; {\rm m}[/tex].For the weight of the student ([tex]784.8\; {\rm N}[/tex]), the lever arm is [tex](3.0 - 2.0)\; {\rm m} = 1.0\; {\rm m}[/tex].

Hence:

[tex]\begin{aligned}N_{\texttt{1}}\, (3.0) = (981)\, (1.5) + (784.8) \, (1.0) \end{aligned}[/tex].

[tex]\begin{aligned}N_{\texttt{1}} &= \frac{(981)\, (1.5) + (784.8) \, (1.0)}{3.0} \; {\rm N} =752.1\; {\rm N}\end{aligned}[/tex].

In other words, support [tex]\texttt{1}[/tex] would exert an upward force of [tex]752.1\; {\rm N}[/tex] on the beam.

at what rate is energy being dissipated as joule heat in the resistor after an elapsed time equal to the time constant of the circuit? answer in units of w.

Answers

The rate at which energy is being dissipated as Joule heat in a resistor can be calculated using the formula [tex]P=I^2R[/tex], and after an elapsed time equal to the time constant of the circuit, the power dissipated by the resistor can be given by [tex]P=0.4I^2 \times R[/tex].

The rate at which energy is being dissipated as Joule heat in a resistor is equal to the power dissipated by the resistor, which can be calculated using the formula [tex]P=0.4I^2\times R[/tex], where P is the power dissipated in watts, I is the current flowing through the resistor in amperes, and R is the resistance of the resistor in ohms.

After an elapsed time equal to the time constant of the circuit, the current flowing through the circuit will have reached approximately 63.2% of its maximum value. This is because the time constant of a circuit is equal to the product of the resistance and the capacitance, and it represents the amount of time it takes for the current in the circuit to reach 63.2% of its maximum value.

At this point, the power dissipated by the resistor can be calculated using the formula [tex]P=0.4I^2 \times R[/tex]. Since the current is 63.2% of its maximum value, we can substitute 0.632I for I in the formula. Therefore, the power dissipated by the resistor at this point is:

P = (0.632*I)^2 * R

= [tex]P=0.4I^2 \times R[/tex]

where I is the maximum current that will flow through the circuit, and R is the resistance of the resistor in ohms.

The rate at which energy is being dissipated as Joule heat in the resistor is equal to the power dissipated by the resistor, which is given by the above equation. Therefore, the answer to the question is:

Rate of energy dissipation = [tex]P=0.4I^2 \times R[/tex] watts

where I is the maximum current that will flow through the circuit, and R is the resistance of the resistor in ohms.

To learn more about resistors

https://brainly.com/question/24297401

#SPJ4

Other Questions
a play in which events are connected to each other in strict, chronological order, with a cause-effect logic, and in which dramatic experience attempts to convey a lifelike progression of experience through time, is classified as continuous in structure and nonlinear in chronology. discontinuous in structure and linear in chronology. not being an imaginative piece of art. continuous in structure and linear in chronology. how does the 179 immediate expensing deduction affect the computation of macrs cost recovery? michael is 3 years old, and while playing outside he falls and scrapes his knee. when his grandmother later asks him what happened, he tells her that the driveway was being mean and made him fall. which limitation of childhood thinking does this belief reflect? Suppose I have the von Neumann-Morgenstern utility function u (C1,C2) = 1/C1 + (1 - 1)2 where it is the probability that state of the world 1 occurs. What is my expected utility from a lottery that pays $64 with probability 0.25 and $16 with probability 0.75? a common way to describe acceleration is to express it in multiples of g, earth's gravitational acceleration. if a dragster accelerates at a rate of 39.2 m/s2, how many g's does the driver experience? consider an economy with a fixed exchange rate and money supply equal to 2 billion pesos. the country has 1 billion in reserves and 1 billion in domestic credit. if there is a sudden decline in the demand for money, then: Which of the following is an example of self-observation?A. Watching another person's actions and judging their choicesB. Determining how you feel about a situation and how to change your thoughtsC. Ignoring the problem in hope it will go awayD. Helping another person to change their thoughts. The ranking of American Indians on social distance scales proves there has been a decline in prejudice toward them since the 1970s. (true or false) the legal health record must meet standards as defined by the followingfederal regulations, state laws, accreditation body standards A firm's bonds have a maturity of 8 years with a $1,000 face value, have an 11% semiannual coupon, are callable in 4 years at $1,154, and currently sell at a price of $1,283.09.What is their nominal yield to maturity? Round your answer to two decimal places.What is their nominal yield to call? Round your answer to two decimal places. %What return should investors expect to earn on these bonds? Suppose a trader would like to buy a t1-maturity bond at time t0. The trader also wants this bond to be liquid. Unfortunately, he discovers that the only bond that is liquid is an on-the-run Treasury with a longer maturity of t2. All other bonds are off-the-run. How can the trader create the liquid short-term bond synthetically assuming that all bonds are of discount type and that, contrary to reality, forward loans are liquid? ( 10 Points) When Acme Dynamite produces 200 units of output, its variable cost is $3,000, and its fixed cost is $500. It sells each unit of output for $30. When Acme Dynamite produces 200 units of output, its profit is A. $2,000 B. $6,000 C. $2,500 D. $3,000 Please Ill give brainliest A Ferris wheel reaches a maximum height of 60 m above the ground and takes twelve minutes to complete one revolution. Riders have to climb a m staircase to board the ride at its lowest point.(a) [4 marks] Write a sine function for the height of Emma, who is at the very top of the ride when t = 0.(b) [2 marks] Write a cosine function for Eva, who is just boarding the ride.(c) (2 marks] Write a sine function for Matthew, who is on his way up, and is at the same height as the central axle of the wheel. a kite flying in the air has a 94- string attached to it, and the string is pulled taut. the angle of elevation of the kite is . find the height of the kite. round your answer to the nearest tenth. Explain what events and actions caused the conflictbetween Barbara Frietchie and the Confederate soldiers.Write three to four sentences. what type of communication is able to pass data in two directions at the same time(input or output)? We learn this one tonite:Computing Taxable Income. Ross Martin arrived at the following tax information:Gross salary, $56,145Interest earnings, $205Dividend income, $65Standard deduction, $12,000Itemized deductions, $11,250Adjustments to income, $1,200What amount would Ross report as taxable income?Formula: Ross would have a taxable income of $43,215 resulting from $56,145 + $205 + $65 - $1,200 - $12,000.Gross Income(wages, profits, dividends, interest, and other income, $56,145 + 205 +65Less: any adjustments to income (i.e. retirement plan contributions) -$1,200Equals: Adjusted Gross Income $55,485Less: itemized deductions (standard deductions) (takes the standard ded.) -$12,000Equals: Taxable Income $43,215Lets say you were a single filers, then you look up your rate on the tax table such as the tax schedule below from IRS and www.NerdWallet.comWhat would be you tax rate (Assume Ross is single):2021 federal income tax brackets(for taxes due in April 2022 or in October 2022 with an extension)Expand the filing status that applies to you.Single filersTax rateTaxable income bracketTax owed10%$0 to $9,95010% of taxable income12%$9,951 to $40,525$995 plus 12% of the amount over $9,95022%$40,526 to $86,375$4,664 plus 22% of the amount over $40,52524%$86,376 to $164,925$14,751 plus 24% of the amount over $86,37532%$164,926 to $209,425$33,603 plus 32% of the amount over $164,92535%$209,426 to $523,600$47,843 plus 35% of the amount over $209,42537%$523,601 or more$157,804.25 plus 37% of the amount over $523,600Question 2, part B: So if you were a single filer and made what Ross made, you fall in the 22% brackets, so how much taxes would you pay?Question 2, part C: What would be your average tax rate?Since we use a progressive tax system, if you earn more, you pay more: Example: Let us say youre a single filer with $32,000 in taxable income. That puts you in the 12% tax bracket in 2021. But do you pay 12% on all $32,000? No. Actually, you pay only 10% on the first $9,950; you pay 12% on the rest. (Look at the tax brackets above to see the breakout.)The United States has a progressive tax system, meaning people with higher taxable incomes pay higher federal income tax rates. Being "in" a tax bracket doesn't mean you pay that federal income tax rate on everything you make. The progressive tax system means that people with higher taxable incomes are subject to higher federal income tax rates, and people with lower taxable incomes are subject to lower federal income tax rates. The government decides how much tax you owe by dividing your taxable income into chunks also known as tax brackets and each chunk gets taxed at the corresponding tax rate. The beauty of this is that no matter which bracket youre in, you wont pay that tax rate on your entire income. (This is the idea behind the concept of an effective tax rate.) How does the author BEST develop the theme that not everything or everyone is as it seems in part one of too kill a mocking bird the command module of the apollo spacecraft can be modelled as a truncated cone with a base diameter of 3.9 meters, a diameter at the upper (apex) end of 1.3 meters and a length of 3.5 meters. suppose the base of the spacecraft is encircled by a metal ring which is an excellent conductor. the magnitude and direction of the earth's magnetic field do not vary significantly over a distance the size of the spacecraft. if the spacecraft is oriented while in orbit so that its long axis is parallel to the earth's magnetic field, which has a magnitude of 1.0 x 10-4 t, and it then rotates about a perpendicular axis, which one of maxwell's equations allows us to calculate how much current will flow in the metal ring? A cohesive marketing mix and the comprise a marketing program, Multiple Choice core competencies organizational structure basic marketing evaluation criteria traditional market related budget